Difference between revisions of "2024 AIME II Problems/Problem 12"

Line 1: Line 1:
:D :D :D
+
Let <math>O(0,0),A(\tfrac{1}{2},0),</math> and <math>B(0,\tfrac{\sqrt{3}}{2})</math> be points in the coordinate plane. Let <math>\mathcal{F}</math> be the family of segments <math>\overline{PQ}</math> of unit length lying in the first quadrant with <math>P</math> on the <math>x</math>-axis and <math>Q</math> on the <math>y</math>-axis. There is a unique point <math>C</math> on <math>\overline{AB},</math> distinct from <math>A</math> and <math>B,</math> that does not belong to any segment from <math>\mathcal{F}</math> other than <math>\overline{AB}</math>. Then <math>OC^2=\tfrac{p}{q}</math>, where <math>p</math> and <math>q</math> are relatively prime positive integers. Find <math>p+q</math>.
 +
 
 +
==Solution 1==
 +
By Furaken
 +
<asy>
 +
pair O=(0,0);
 +
pair X=(1,0);
 +
pair Y=(0,1);
 +
pair A=(0.5,0); pair B=(0,sin(pi/3));
 +
dot(O);
 +
dot(X);
 +
dot(Y); dot(A); dot(B);
 +
draw(X--O--Y);
 +
draw(A--B);
 +
label("$B'$", B, W);
 +
label("$A'$", A, S);
 +
label("$O$", O, SW);
 +
pair C=(1/8,3*sqrt(3)/8);
 +
dot(C);
 +
pair D=(1/8,0); dot(D);
 +
pair E=(0,3*sqrt(3)/8); dot(E);
 +
label("$C$", C, NE); label("$D$", D, S); label("$E$", E, W);
 +
draw(D--C--E);
 +
</asy>
 +
 
 +
Let <math>C = (\tfrac18,\tfrac{3\sqrt3}8)</math>. Draw a line through <math>C</math> intersecting the <math>x</math>-axis at <math>A'</math> and the <math>y</math>-axis at <math>B'</math>. We shall show that <math>A'B' \ge 1</math>, and that equality only holds when <math>A'=A</math> and <math>B'=B</math>.
 +
 
 +
Let <math>\theta = \angle OA'C</math>. Draw <math>CD</math> perpendicular to the <math>x</math>-axis and <math>CE</math> perpendicular to the <math>y</math>-axis as shown in the diagram. Then
 +
<cmath>8A'B' = 8CA' + 8CB' = \frac{3\sqrt3}{\sin\theta} + \frac{1}{\cos\theta}</cmath>
 +
By some inequality (i forgor its name),
 +
<cmath>\left(\frac{3\sqrt3}{\sin\theta} + \frac{1}{\cos\theta}\right) \cdot \left(\frac{3\sqrt3}{\sin\theta} + \frac{1}{\cos\theta}\right) \cdot (\sin^2\theta + \cos^2\theta) \ge (3+1)^3 = 64</cmath>
 +
We know that <math>\sin^2\theta + \cos^2\theta = 1</math>. Thus <math>\tfrac{3\sqrt3}{\sin\theta} + \tfrac{1}{\cos\theta} \ge 8</math>. Equality holds if and only if
 +
<cmath>\frac{3\sqrt3}{\sin\theta} : \frac{1}{\cos\theta} = \frac{3\sqrt3}{\sin\theta} : \frac{1}{\cos\theta} = \sin^2\theta : \cos^2\theta</cmath>
 +
which occurs when <math>\theta=\tfrac\pi3</math>. Guess what, <math>\angle OAB</math> ''happens'' to be <math>\tfrac\pi3</math>, thus <math>A'=A</math> and <math>B'=B</math>. Thus, <math>AB</math> is the only segment in <math>\mathcal{F}</math> that passes through <math>C</math>. Finally, we calculate <math>OC^2 = \tfrac1{64} + \tfrac{27}{64} = \tfrac7{16}</math>, and the answer is <math>\boxed{023}</math>.

Revision as of 06:07, 9 February 2024

Let $O(0,0),A(\tfrac{1}{2},0),$ and $B(0,\tfrac{\sqrt{3}}{2})$ be points in the coordinate plane. Let $\mathcal{F}$ be the family of segments $\overline{PQ}$ of unit length lying in the first quadrant with $P$ on the $x$-axis and $Q$ on the $y$-axis. There is a unique point $C$ on $\overline{AB},$ distinct from $A$ and $B,$ that does not belong to any segment from $\mathcal{F}$ other than $\overline{AB}$. Then $OC^2=\tfrac{p}{q}$, where $p$ and $q$ are relatively prime positive integers. Find $p+q$.

Solution 1

By Furaken [asy] pair O=(0,0); pair X=(1,0); pair Y=(0,1); pair A=(0.5,0); pair B=(0,sin(pi/3)); dot(O); dot(X); dot(Y); dot(A); dot(B); draw(X--O--Y); draw(A--B); label("$B'$", B, W); label("$A'$", A, S); label("$O$", O, SW); pair C=(1/8,3*sqrt(3)/8); dot(C); pair D=(1/8,0); dot(D); pair E=(0,3*sqrt(3)/8); dot(E); label("$C$", C, NE); label("$D$", D, S); label("$E$", E, W); draw(D--C--E); [/asy]

Let $C = (\tfrac18,\tfrac{3\sqrt3}8)$. Draw a line through $C$ intersecting the $x$-axis at $A'$ and the $y$-axis at $B'$. We shall show that $A'B' \ge 1$, and that equality only holds when $A'=A$ and $B'=B$.

Let $\theta = \angle OA'C$. Draw $CD$ perpendicular to the $x$-axis and $CE$ perpendicular to the $y$-axis as shown in the diagram. Then \[8A'B' = 8CA' + 8CB' = \frac{3\sqrt3}{\sin\theta} + \frac{1}{\cos\theta}\] By some inequality (i forgor its name), \[\left(\frac{3\sqrt3}{\sin\theta} + \frac{1}{\cos\theta}\right) \cdot \left(\frac{3\sqrt3}{\sin\theta} + \frac{1}{\cos\theta}\right) \cdot (\sin^2\theta + \cos^2\theta) \ge (3+1)^3 = 64\] We know that $\sin^2\theta + \cos^2\theta = 1$. Thus $\tfrac{3\sqrt3}{\sin\theta} + \tfrac{1}{\cos\theta} \ge 8$. Equality holds if and only if \[\frac{3\sqrt3}{\sin\theta} : \frac{1}{\cos\theta} = \frac{3\sqrt3}{\sin\theta} : \frac{1}{\cos\theta} = \sin^2\theta : \cos^2\theta\] which occurs when $\theta=\tfrac\pi3$. Guess what, $\angle OAB$ happens to be $\tfrac\pi3$, thus $A'=A$ and $B'=B$. Thus, $AB$ is the only segment in $\mathcal{F}$ that passes through $C$. Finally, we calculate $OC^2 = \tfrac1{64} + \tfrac{27}{64} = \tfrac7{16}$, and the answer is $\boxed{023}$.